CVS Healio Cases Flashcards

(148 cards)

1
Q

An 87-year-old woman is admitted with pneumonia,
complicated by septic shock. She develops atrial fibrillation with an
uncontrolled ventricular response. Her temperature is 101.8 F, blood pressure
80/40 mm Hg, respirations 20 per minute and heart rate 148 beats per minute.
Prior to the atrial fibrillation, her blood pressure was similarly low. Which
of the following is the next appropriate step in management?
A. Intravenous metoprolol
B. Intravenous diltiazem
C. Intravenous amiodarone
D. Emergent direct current cardioversion
E. Emergent chemical cardioversion with ibutilide

A

Correct Answer: C. Intravenous amiodarone

How well did you know this?
1
Not at all
2
3
4
5
Perfectly
2
Q

Explanation: When a patient is hypotensive due to atrial fibrillation

A

then emergent cardioversion is necessary; however

How well did you know this?
1
Not at all
2
3
4
5
Perfectly
3
Q

A 24-year-old female presents with dyspnea and edema. She
is noted to have a holosystolic murmur at the left lower sternal border as well
as a short early diastolic murmur at the same location. An echocardiogram
reveals the presence of a supracristal ventricular septal defect. Which of the
following explains the diastolic murmur?
A. Aortic regurgitation
B. Mitral stenosis
C. Cor triatriatum
D. Ebstein’s anomaly
E. Pulmonic regurgitation

A

Correct Answer: A. Aortic regurgitation

How well did you know this?
1
Not at all
2
3
4
5
Perfectly
4
Q

Explanation: Supracristal ventricular septal defects occur just beneath the aortic valve at the left ventricular outflow tract. A Venturi effect can occur from the left to right shunt

A

causing the aortic valve leaflet to prolapse into the ventricular septal defect and resulting in significant aortic valve regurgitation.

How well did you know this?
1
Not at all
2
3
4
5
Perfectly
5
Q

A 57-year-old man with no cardiovascular past medical
history presents with acute onset palpitations and intermittent dizziness. He
presents to the emergency department 8 hours after the onset of symptoms and
has never had symptoms like this before. He is found to have atrial
fibrillation with an uncontrolled ventricular response (heart rate 170 beats
per minute). An ECG is completely normal, as is laboratory evaluation with a
basic metabolic panel, complete blood count and a thyroid stimulating hormone
level. Which of the following is a reasonable strategy?
A. Direct current cardioversion with aspirin alone for
thromboembolism prophylaxis
B. Direct current cardioversion with full anticoagulation
for 3 to 4 weeks afterwards for thromboembolism prophylaxis
C. Transesophageal echocardiography to exclude a left atrial
appendage thrombus followed by direct current cardioversion and aspirin alone
for thromboembolism prophylaxis
D. Transesophageal echocardiography to exclude a left atrial
appendage thrombus followed by direct current cardioversion and full
anticoagulation for 3 to 4 weeks afterwards for thromboembolism prophylaxis

A

Correct Answer: A. Direct current cardioversion with aspirin alone for

How well did you know this?
1
Not at all
2
3
4
5
Perfectly
6
Q

thromboembolism prophylaxis

A
How well did you know this?
1
Not at all
2
3
4
5
Perfectly
7
Q

Explanation: When the duration of atrial fibrillation is thought to be less than 48 hours

A

transesophageal echocardiography is not required to evaluate for left atrial appendage thrombus. Direct current cardioversion can safely be performed with very low risk for thromboembolism in this situation.

How well did you know this?
1
Not at all
2
3
4
5
Perfectly
8
Q

A 58-year-old woman with asthma and a history of
hypertension, type 2 diabetes and a prior stroke presents to the emergency
department with substernal chest pains radiating to her left arm. She is short
of breath, diaphoretic and nauseated. She is afebrile with a blood pressure of
150/90 mm Hg, heart rate of 90 beats per minute, respirations 22 per minute and
oxygen 92% on room air. Physical examination reveals an S4 heart sound and
significant wheezing on lung examination. Her ECG is below:
Enlarge

Image: Learn the Heart
Which of the following combinations of initial medical
therapy is appropriate?
A. Aspirin, clopidogrel, unfractionated heparin,
beta-blocker, oxygen, nitroglycerin
B. Aspirin, prasugrel, low molecular weight heparin,
non-dihydropyridine calcium channel blocker, oxygen, nitroglycerin
C. Aspirin, clopidogrel, low molecular weight heparin,
non-dihydropyridine calcium channel blocker, oxygen, nitroglycerin
D. Aspirin, prasugrel, unfractionated heparin, beta-blocker,
oxygen, nitroglycerin

A

Correct Answer: C. Aspirin

How well did you know this?
1
Not at all
2
3
4
5
Perfectly
9
Q

non-dihydropyridine calcium channel blocker

A

oxygen

How well did you know this?
1
Not at all
2
3
4
5
Perfectly
10
Q

Explanation: The non-dihydropyridine calcium channel blockers diltiazem and verapamil can be used when there is a contraindication to beta-blockers (such as asthma) and there is no heart failure or significant left ventricular systolic dysfunction present. They are especially helpful to lower heart rate and reduce oxygen demand in this situation. Sublingual nifedipine is contraindicated due to a reflexive increase in the sympathetic nervous system

A

which can be harmful.

How well did you know this?
1
Not at all
2
3
4
5
Perfectly
11
Q

Recall that prasugrel is contraindicated if a prior stroke or transient ischemic attack is present. Also

A

either low molecular weight heparin or unfractionated heparin are considered reasonable choices for anticoagulation.

How well did you know this?
1
Not at all
2
3
4
5
Perfectly
12
Q

A 58-year-old man is undergoing a treadmill ECG stress
test for evaluation of chest pain. After 6 minutes on a Bruce protocol, he
develops substernal chest pain radiating to his left arm and there is 4 mm of
ST depression in leads V5 and V6. His vital signs are stable. Which of the
following would you expect to find on his physical examination?
A. An S3 heart sound and a systolic ejection murmur
B. An S3 heart sound and a holosystolic murmur at the
cardiac apex
C. An S4 heart sound and a systolic ejection murmur
D. An S4 heart sound and a holosystolic murmur at the
cardiac apex

A

Correct Answer: D. An S4 heart sound and a holosystolic murmur at the

How well did you know this?
1
Not at all
2
3
4
5
Perfectly
13
Q

cardiac apex

A
How well did you know this?
1
Not at all
2
3
4
5
Perfectly
14
Q

Explanation: Physical examination findings are relatively non-specific usually only present during the anginal episode making this a less helpful means of diagnosis. When examined during an anginal attack

A

the heart rate and blood pressure may be elevated due to increased sympathetic tone. An S4 heart sound may be present during myocardial ischemia due to the lack of adenosine triphosphate production impairing left ventricular relaxation. Recall that myocardial relaxation is an active process requiring adenosine triphosphatewhich is reduced during ischemia and a S4 heart sound occurs when a non-compliant

How well did you know this?
1
Not at all
2
3
4
5
Perfectly
15
Q

During inferior ischemia

A

posteromedial papillary muscle dysfunction can cause mitral regurgitation resulting in a holosystolic murmur at the cardiac apex radiating to the axilla. This rarely occurs during anterior or lateral ischemia since the anterolateral papillary muscle has dual supply from the left anterior descending and circumflex coronary artery.

How well did you know this?
1
Not at all
2
3
4
5
Perfectly
16
Q

When the left ventricular end-diastolic pressure increases during myocardial ischemia

A

that pressure can be transmitted backward to the pulmonary veins and into the pulmonary vasculature causing transient pulmonary edema resulting in dyspnea and rales on lung examination.

How well did you know this?
1
Not at all
2
3
4
5
Perfectly
17
Q

A 38-year-old woman presents with the acute onset of
palpitations. Her 12-lead ECG is below. She is afebrile with a heart rate of
180 beats per minute, blood pressure of 120/80 mm Hg and respirations of 18 per
minute. Which of the following is an appropriate course of action?
Enlarge

A. Emergent direct current cardioversion
B. Adenosine 6 mg intravenously, followed by a saline flush
C. Intravenous procainamide
D. Intravenous amiodarone

A

Correct Answer: B. Adenosine 6 mg intravenously

How well did you know this?
1
Not at all
2
3
4
5
Perfectly
18
Q

Explanation: When a narrow QRS complex tachycardia is present

A

and the diagnosis is unclear

How well did you know this?
1
Not at all
2
3
4
5
Perfectly
19
Q

There is no need for emergent direct current cardioversion

A

as the patient is hemodynamically stable. Antiarrhythmic drug therapy is not recommended until the rhythm is diagnosed. Procainamide is not commonly used

How well did you know this?
1
Not at all
2
3
4
5
Perfectly
20
Q

A 58-year-old man presents to the emergency department
after experiencing intermittent chest pain for 2 days. He describes substernal
chest pressure radiating to his jaw. He is short of breath and diaphoretic. His
temperature is 100 F, blood pressure is 140/90 mm Hg, respirations are 22
breaths per minute, heart rate is 70 beats per minute and oxygen is 92% on room
air.
He is taken for emergency percutaneous coronary
intervention. This discovers thrombus in the mid-left anterior descending
coronary artery, and primary percutaneous coronary intervention is performed;
however, there was sluggish forward flow afterwards despite the left anterior
descending artery remaining widely patent.
After a diagnosis ofanterior ST-elevation
myocardial infarction complicated by no-reflow, he is followed and discharged
home. During a clinic follow-up visit 6 weeks later, he is diagnosed with left
ventricular aneurysm.
Four months later, he calls 911 due to sudden onset of
weakness and pre-syncope. He is short of breath, diaphoretic and feels
significant palpitations. He is afebrile with a blood pressure of 80/40 mm Hg,
heart rate of 240 beats per minute, respirations 26 breaths per minute and
oxygen 88% on room air. His ECG is below:
Enlarge

Image: Learn the Heart
Which of the following is the correct diagnosis?
A. Ventricular fibrillation
B. Monomorphic ventricular tachycardia
C. Polymorphic ventricular tachycardia
D. Wolff-Parkinson-White with pre-excitation and atrial
fibrillation

A

Correct Answer: B. Monomorphic ventricular tachycardia

How well did you know this?
1
Not at all
2
3
4
5
Perfectly
21
Q

Explanation: See theVentricular Tachycardia ECG Reviewfor a complete summary.

A
How well did you know this?
1
Not at all
2
3
4
5
Perfectly
22
Q

The four main complications of a left ventricular aneurysm include the following:

A
How well did you know this?
1
Not at all
2
3
4
5
Perfectly
23
Q

Ventricular tachycardia: The scar within the left ventricular aneurysm is a focus for ventricular arrhythmias which can lead to sudden cardiac death.

A
How well did you know this?
1
Not at all
2
3
4
5
Perfectly
24
Q

Heart failure: The portion of the heart that contains the aneurysm is not contractile and is frequently “dyskinetic.” This results in overall decrease in heart function and the development of congestive heart failure.

A
How well did you know this?
1
Not at all
2
3
4
5
Perfectly
25
Left ventricular thrombus formation: When blood stagnates in any area of the body
there is a risk of platelet aggregation and thrombus formation. The aneurysmal portion of the left ventricle is no different. Embolization of left ventricular thrombi can lead to embolic stroke or other systemic embolisms.
26
Angina pectoris: The aneurysmal tissue can still cause symptoms of angina
even if revascularized.
27
There is a surgical procedure during which the surgeon resects the aneurysm and uses a Dacron patch. This is called the “Dor procedure” or the endoventricular circular patch plasty. This procedure is indicated when medical therapy fails to control or acceptably improve the above mentioned complications/symptoms from the left ventricular aneurysm.
28
A 21-year-old man has been progressively short of breath and fatigued. He has not seen a physician since childhood. Physical examination reveals a loud IV/VI systolic ejection murmur at the left lower sternal border much louder with Valsalva as well as a mid-systolic murmur at the cardiac apex. An echocardiogram shows an interventricular septal thickness of 35 mm. Which of the following is the likely cause of the murmur at the cardiac apex? A. Left ventricular outflow tract obstruction B. Tricuspid regurgitation C. Austin-Flint murmur D. Venturi effect E. A ventricular septal defect
Correct Answer: D. Venturi effect
29
Explanation: This patient has hypertrophic obstructive cardiomyopathy
or HOCM
30
The Venturi effect is described as a decrease in pressure when blood flows through a stenosis at a high velocity. This relates to the hemodynamics in HOCM
in which the increased velocity in the left ventricular outflow tract decreases pressure in this region causing the anterior leaflet of the mitral valve to be pulled in that direction. This is called “systolic anterior motion
31
A 58-year-old woman presents with chest pains. She has had no recent bleeding or trauma and has no history of stroke. She is afebrile with a heart rate of 70 beats per minute, blood pressure 140/90 mm Hg and respirations 20 per minute. Her ECG is below: Enlarge  Image: Learn the Heart She is given aspirin, clopidogrel, metoprolol and unfractionated heparin. Nitrates have completely relieved her chest pains. Her troponin measured 15 mg/L. Which of the following is an appropriate treatment strategy? A. Thrombolytic therapy B. Initial conservative therapy with medical management C. Early invasive angiography
Correct Answer: C. Early invasive angiography
32
Explanation: An early invasive strategy refers to proceeding to coronary angiography with possible percutaneous coronary intervention (percutaneous coronary intervention or coronary stenting) within 4 to 24 hours of hospital admission. An initial conservative management consists of medical therapy only without plans to proceed to coronary angiography and percutaneous coronary intervention.
33
The following factors would warrant an early invasive strategy:
34
Increased cardiac biomarkers (troponin
creatine kinase-myocardial band)
35
New ST segment depression
36
Signs or symptoms of congestive heart failure (rales on examination
hypoxia with pulmonary edema on chest X-ray)
37
Hemodynamic instability
38
Sustained ventricular tachycardia or ventricular fibrillation
39
Recent coronary intervention within 6 months
40
Prior coronary artery bypass grafting
41
High Thrombolysis in Myocardial Infarction risk score
42
Reduced left ventricular systolic function (ejection fraction < 40%)
43
Recurrent angina at rest or with low-level activity
44
High-risk findings from non-invasive testing
45
The ICTUS trial showed no difference in the above approaches in 3 years.
46
The RITA-3 trial showed no difference at 1 year
but there was a reduction of death or myocardial infarction at 5 years in the early invasive arm ― mainly in high-risk patients
47
A 58-year-old man presents to the emergency department after experiencing intermittent chest pain for 2 days. He describes substernal chest pressure radiating to his jaw. He is short of breath and diaphoretic. His temperature is 100 F, blood pressure is 140/90 mm Hg, respirations are 22 breaths per minute, heart rate is 70 beats per minute and oxygen is 92% on room air. He is taken for emergency percutaneous coronary intervention. This discovers thrombus in the mid-left anterior descending coronary artery, and primary percutaneous coronary intervention is performed; however, there was sluggish forward flow afterwards despite the left anterior descending artery remaining widely patent. After a diagnosis of anterior ST-elevation myocardial infarction complicated by no-reflow, he is followed and discharged home. During a clinic follow-up visit 6 weeks later, the below ECG is obtained. Enlarge  Image: Learn the Heart Which of the following is the correct diagnosis? A. Pericarditis B. Normal ECG changes after an anterior myocardial infarction C. Left ventricular aneurysm D. Left bundle branch block
Correct Answer: C. Left ventricular aneurysm
48
Explanation: A left ventricular aneurysm can form after a ST-elevation myocardial infarction. Most commonly
the apex of the heart is involved; however
49
Without knowing a patient's past medical history
the ECG changes of an aneurysm may mimic an acute ST segment elevation myocardial infarction. With an anterior or apical aneurysm
50
A 55-year-old man presents with acute onset chest pain and diaphoresis. His ECG reveals an acute myocardial infarction, and emergency intervention is performed. He is treated medically and discharged home in stable condition. Five days later, he is found nearly unresponsive and brought to the ED. His temperature is 95, blood pressure 70 mm Hg systolic/30 mm Hg diastolic, heart rate 120 beats per minute and respirations 30 per minute. No murmur is appreciated, and examination otherwise reveals rales throughout both lung fields. Repeat angiography shows a patent stent and hyperdynamic left ventricular systolic function, but there are large V waves in the pulmonary capillary wedge pressure tracing. Which of the following is the most likely diagnosis? A. Acute ventricular septal defect B. Free wall rupture causing cardiac tamponade C. Acute mitral regurgitation D. Aortic rupture
Correct Answer: C. Acute mitral regurgitation
51
Explanation: Acute mitral valve regurgitation can occur as a complication of a myocardial infarction from papillary muscle rupture. When a large pressure is forced into the left atrium from the mitral regurgitant volume during systole
a large pressure wave is created in the pulmonary capillary wedge pressure tracing. This is termed the V wave. Normal V waves are small but become quite large with severe mitral regurgitation.
52
Frequently
no murmur is heard on examination in the setting of acute mitral regurgitation because the pressure in the left ventricle and left atrium equalize quickly (the left atrium has not had time to remodel and dilate). At times
53
A 53-year-old man with type 2 diabetes has an inferior myocardial infarction. His ejection fraction remains 30% afterward. He then develops atrial fibrillation with an uncontrolled ventricular response. He is rate-controlled on carvedilol and digoxin but continues to have significant palpitations, dyspnea and fatigue despite optimal heart failure therapy and coronary revascularization. An electrical cardioversion was not successful to maintain sinus rhythm to relieve his symptoms. Amiodarone is initiated; however, he has significant side effects and is not able to tolerate this therapy. Which of the following antiarrhythmic drug therapies would be considered appropriate for this patient? A. Dofetilide B. Sotalol C. Dronedarone D. Flecainide E. Procainamide
Correct Answer: A. Dofetilide
54
Explanation: Remember that amiodarone and dofetilide are the only two antiarrhythmic drugs considered safe in the setting of left ventricular systolic function.
55
A 66-year-old woman presents to the emergency department with chest pain, shortness of breath, nausea and vomiting. She has had symptoms for 3 days and thought it was indigestion; however, it is now worse. Her temperature is 99.2 F, blood pressure 140/90 mm Hg, heart rate 70 beats per minute, respirations 20 per minute and oxygen normal on room air. She has an S4 heart sound, but no murmurs are appreciable. Her ECG is below. Enlarge  Image: Learn the Heart She is treated appropriately for an acute inferior wall myocardial infarction with aspirin, clopidogrel, heparin, a beta-blocker, oxygen and percutaneous coronary intervention of the right coronary artery. On day two of her hospitalization, she becomes acutely short of breath. She remained afebrile with a heart rate of 90 beats per minute, respirations 36 per minute and blood pressure 80/40 mm Hg. Physical examination revealed once again an S4 heart sound and no murmur. Her pulsus paradoxus is 8 mm Hg. A chest X-ray showed pulmonary edema. She is brought back for coronary angiography and her stent in the right coronary artery remains patent. Right heart catheterization reveals a large V wave in the pulmonary capillary wedge tracing. The oxygen saturation measured in the right atrium was 65%, right ventricle 66% and pulmonary artery 65%. Which of the following is the correct diagnosis? A. Acute ventricular septal defect B. Acute left ventricular free wall rupture C. Acute mitral valve regurgitation D. Right ventricular infarction
Correct Answer: C. Acute mitral valve regurgitation
56
Explanation: The two complications of a myocardial infarction that can cause hypotension with pulmonary edema are an acute ventricular septal defect or acute mitral valve regurgitation.
57
Acute severe mitral regurgitation is a life-threatening disorder. Papillary muscle rupture after acute myocardial infarction can occur as a complication of an inferior myocardial infarction (right coronary artery supply)
as the posteromedial papillary muscle is the most likely to rupture.
58
There are two papillary muscles that comprise part of the complex anatomy of the mitral valve. The anterolateral papillary muscle receives dual blood supply from the left anterior descending coronary artery and the left circumflex coronary artery in most individuals
whereas the posteromedial papillary muscle receives its sole blood supply from the right coronary artery. Complete infarction of the posteromedial papillary muscle can occur during an inferior myocardial infarction
59
When a large pressure is forced into the left atrium during systole from the mitral regurgitant volume
a large pressure wave is created
60
Recall that acute mitral regurgitation may not cause a murmur. The pressures of the left ventricle and left atrium equalize very quickly in systole
as there has been no time for the chambers to adapt to the acute change (dilate). This results in a very short duration of turbulent flow backward through the mitral valve
61
Emergent surgical repair or replacement of the mitral valve is indicated for acute mitral regurgitation. Mortality approaches 100% if not surgically fixed.
62
An acute ventricular septal defect does not cause large V waves and frequently does create a holosystolic murmur. An acute free wall rupture results in cardiac tamponade from a rapidly accumulating pericardial effusion. This increases the pulsus paradoxus to greater than 12 mm Hg. Right ventricular infarction is a complication of inferior myocardial infarction and can cause shock
but not pulmonary edema. Again
63
A 32-year-old woman presents to the emergency department after a syncopal episode. She is experiencing palpitations and is dizzy. Her blood pressure is 90/60 mm Hg and her heart rate is 220 beats per minute. Her ECG strip is below: Enlarge  What is the initial appropriate medical management? A. Intravenous diltiazem B. Intravenous metoprolol C. Intravenous procainamide D. Intravenous digoxin
Correct Answer: C. Intravenous procainamide
64
Explanation: The ECG reveals atrial fibrillation with “pre-excitation
” which occurs in the setting of Wolff-Parkinson-White syndrome. Emergent direct current cardioversion would be reasonable if a patient is definitely hemodynamically unstable; however
65
The combination of atrial fibrillation and Wolff-Parkinson-White syndrome can be fatal due to rapid conduction of the atrial activity through the accessory pathway
resulting in rapid ventricular rates causing ventricular fibrillation. Atrioventricular nodal blocking agents (diltiazem
66
When atrial fibrillation is seen in a patient with Wolff-Parkinson-White
ablation of the accessory pathway is recommended to prevent future rapid conduction to the ventricles.
67
A 45-year-old man with a history of dilated cardiomyopathy presents with increased dyspnea and palpitations. He is known to have an ejection fraction of 20% and is already taking carvedilol 25 mg orally twice daily. He is afebrile with a heart rate of 140 beats per minute, blood pressure 110/60 mm Hg and respirations 20 per minute. He has elevated jugular venous pressure, or JVP, and rales on lung examination. His laboratory evaluation is normal. Which of the following would be best to either lower heart rate or maintain sinus rhythm in this case? A. Diltiazem B. Dronedarone C. Digoxin D. Sotalol E. Propafenone
Correct Answer: C. Digoxin
68
Explanation: Digoxin is best indicated to control heart rate in patients with atrial fibrillation with known left ventricular systolic dysfunction (class I indication). Digoxin receives a class IIa indication to be used in combination with beta-blockers and/or nondihydropyridine calcium channel blockers in any situation to control the heart rate in atrial fibrillation. There is a IIb indication to control the heart rates at rest in the presence of persistent atrial fibrillation and a class III (not recommended
may cause harm) indication to use digoxin as the sole agent to treat paroxysmal atrial fibrillation. Digoxin toxicity is always a concern.
69
An 84-year-old man has undergone bypass surgery on three occasions. All of his saphenous vein grafts are occluded and he has anginal symptoms with minimal exertion. He is not considered to be a revascularization candidate. He is afebrile, with a heart rate of 55 beats per minute, respirations 20 breaths per minute and blood pressure 110/70 mm Hg. He is taking isosorbide mononitrate 240 mg orally daily, atenolol 100 mg orally daily, amlodipine 10 mg orally daily, ranolazine 1,000 mg orally twice daily, aspirin and simvastatin. Which of the following is the next most appropriate course of action? A. Heart transplant evaluation B. Enhanced external counterpulsation C. Chelation therapy D. Add doxazosin E. Insert a left ventricular assist device
Correct Answer: B. Enhanced external counterpulsation
70
Explanation: Enhanced external counterpulsation is used in the outpatient setting in patients refractory to medical therapy. The technique consists of ECG-gated rapid sequential compression of the lower extremities during diastole followed by simultaneous decompression during systole. Although the exact mechanism(s) of action of enhanced external counterpulsation remains unclear
significant anginal relief has been shown in small studies (MUST-EECP trial).
71
Left ventricular assist devices are not approved for treatment of refractory angina. Left ventricular assist devices are most often used in the management of refractory heart failure. Chelation therapy has a class III indication for the treatment of angina and coronary disease
ie
72
A 38-year-old previously healthy man presents with acute onset palpitations and dizziness 8 hours prior to emergency department arrival. Atrial fibrillation with an uncontrolled ventricular response is present. He is afebrile with a heart rate of 140 beats per minute, blood pressure 120/80 mm Hg and respirations 20 per minute. Laboratory evaluation is normal and a transthoracic echocardiogram is normal. Which of the following is an appropriate management strategy? A. Ibutilide infusion for chemical cardioversion B. Transesophageal echocardiography followed by direct current cardioversion C. Intravenous digoxin alone D. Oral amiodarone
Correct Answer: A. Ibutilide infusion for chemical cardioversion
73
Explanation: Ibutilide is a class III anti-arrhythmic drug underutilized as a means of chemical cardioversion. As long as electrolytes are normal
the baseline corrected QT interval is normal
74
Since this patient has a CHA2DS2-VASc score of 0 and presented within 48 hours
a transesophageal echocardiogram is not indicated to exclude left atrial appendage thrombus. Digoxin alone is never recommended to control atrial fibrillation. Amiodarone is not a good option in young individuals when there are alternative antiarrhythmic drug options due to amiodarone toxicity.
75
A 79-year-old woman with chronic hypertension presents with palpitations and dizziness. She is found to be in atrial fibrillation with an uncontrolled ventricular response (heart rate 160 beats per minute). Her blood pressure is 132/82 mm Hg, respirations 18 per minute and she is afebrile. Physical examination is normal, except for the irregularly irregular heart rhythm and tachycardia. Which of the following is the appropriate next step in management? A. Intravenous amiodarone B. Emergent direct current cardioversion C. Intravenous digoxin D. Intravenous diltiazem or metoprolol E. Flecainide 300 mg orally once
Correct Answer: D. Intravenous diltiazem or metoprolol
76
Explanation: The initial management in a hemodynamically-stable patient with uncontrolled heart rates from atrial fibrillation is a nondihydropyridine calcium channel blockers (diltiazem
verapamil) or beta-blocker. If heart failure is present
77
Amiodarone to restore sinus rhythm (rhythm control strategy) is not appropriate in a stable patient when rate controlling has not even been attempted. This drug has many toxicities and should be avoided if possible.
78
Emergent direct current cardioversion is reserved for hemodynamically-unstable patients (hypotension
chest pain
79
Digoxin alone is not recommended for initial therapy due to less efficacy to lower the heart rate and possible toxicities. If systolic dysfunction is present
it would be a reasonable choice. Digoxin is also frequently used in combination with nondihydropyridine calcium channel blockers (diltiazem
80
Flecainide 300 mg orally once can be used in “lone atrial fibrillation” patients (no structural heart disease or coronary artery disease);
81
A 26-year-old man who is an athletic runner experiences palpitations after a 1 mile race. He is found to have atrial fibrillation with an uncontrolled heart rate. He frequently feels nauseated and flushed after sprinting or after races. Which of the following medications is classically used to prevent future episodes of this arrhythmia? A. Amiodarone B. Bretylium C. Sotalol D. Disopyramide
Correct Answer: D. Disopyramide
82
Explanation: The patient has vagally-mediated atrial fibrillation. The classic therapy is disopyramide. Atrial fibrillation triggered by situations of vagal stimulation has been well described (nausea
vomiting
83
A 55-year-old man presents with acute onset chest pain and diaphoresis. His ECG reveals an acute myocardial infarction, and emergency intervention is performed. He is treated medically and discharged home in stable condition. Five days later, he is found nearly unresponsive and brought to the ED. His temperature is 95, blood pressure 70 mm Hg systolic/30 mm Hg diastolic, heart rate 120 beats per minute and respirations 30 per minute. No murmur is appreciated, and examination otherwise reveals rales throughout both lung fields. Repeat angiography shows a patent stent and hyperdynamic left ventricular systolic function, but there are large V waves in the pulmonary capillary wedge pressure tracing. Which of the following coronary artery and papillary muscle combination is most commonly involved? A. Left anterior descending coronary artery and the anterolateral papillary muscle B. Left circumflex coronary artery and the anterolateral papillary muscle C. Left anterior descending coronary artery and the posteromedial papillary muscle D. Right coronary artery and the posteromedial papillary muscle
Correct Answer: D. Right coronary artery and the posteromedial papillary
84
muscle
85
Explanation: Papillary muscle rupture after acute myocardial infarction most commonly occurs as a complication of an inferior MI (right coronary artery supply)
as the posteromedial papillary muscle is the most likely to rupture.
86
There are two papillary muscles that comprise part of the complex anatomy of the mitral valve. The anterolateral papillary muscle receives dual blood supply from the left anterior descending coronary artery and the left circumflex coronary artery in most individuals
whereas the posteromedial papillary muscle receives its sole blood supply from the right coronary artery. Complete infarction of the posteromedial papillary muscle can occur during an inferior MI (from thrombosis of the right coronary artery)
87
A 52-year-old man presents for a routine physical examination, which reveals a III/VI systolic ejection murmur with a systolic ejection click and a soft S2 heart sound. He has no dyspnea or chest pains and has never had a syncopal episode. Echocardiography reveals aortic stenosis with a mean pressure gradient of 35 mm Hg with an aortic valve area of 1.2 cm2. The aortic root was measured at 5.2 cm. Which of the following is the recommended approach in this situation? A. Start an angiotensin-converting enzyme inhibitors, and repeat an echocardiogram in 1 year or earlier if symptoms develop B. Start a beta-blocker, and repeat an echocardiogram in 1 year or earlier if symptoms develop C. Start a HMG-CoA reductase inhibitor, and repeat an echocardiogram in 1 year or earlier if symptoms develop D. No medical therapy and repeat an echocardiogram in 1 year or earlier if symptoms develop E. Surgical aortic valve replacement and repair of the ascending aortic aneurysm
Correct Answer: E. Surgical aortic valve replacement and repair of the
88
ascending aortic aneurysm
89
Explanation: When the aortic root or ascending aorta reaches greater than 5.0 cm
then surgical repair of the aneurysm is recommended. If the aortic stenosis is at least moderate in severity
90
A 45-year-old woman has coronary artery disease and asthma. She underwent coronary artery bypass grafting for left main coronary stenosis. She is taking diltiazem hydrochloride 120 mg once daily and oral pravastatin 10 mg once a day at bedtime. She is allergic to aspirin. Which of the following is the best course of action? A. Allergist referral for aspirin desensatization B. Add clopidogrel C. Add ticlopidine D. No change in therapy is needed
Correct Answer: B. Add clopidogrel
91
Explanation: Every patient with documented coronary artery disease should be taking antiplatelet therapy
usually in the form of aspirin
92
When a high risk patient is allergic to aspirin
an allergist can “desensitize” the patient so that the allergy is no longer present. This is used only if no alternative to aspirin exists (clopidogrel allergy as well).
93
A 25-year-old woman who is 33 weeks into her pregnancy is becoming increasingly short of breath with some lower-extremity edema. She is afebrile with a heart rate of 110 beats per minute, respirations 20 per minute and blood pressure 100 mm Hg systolic/60 mm Hg diastolic. Physical examination reveals a II/IV early diastolic decrescendo murmur at the cardiac apex. Which of the following is the likely diagnosis? A. Aortic stenosis B. Aortic regurgitation (with Austin-Flint murmur) C. Mitral stenosis D. Mitral regurgitation E. Atrial septal defect
Correct Answer: C. Mitral stenosis
94
Explanation: Pregnant women do not tolerate stenotic valvular problems very well
due to the required increase in cardiac output needed in this state. Regurgitant valve problems are very well-tolerated
95
An 82-year-man is experiencing substernal chest pressure radiating to his left arm with exertion relieved with rest. There is associated shortness of breath. His physical examination reveals an S4 heart sound. There is a III/VI late-peaking systolic ejection murmur at the right upper sternal border with a soft S2 heart sound. There is also a III/VI holosystolic murmur at the apex. His ECG is below (shows left ventricular hypertrophy with strain). A coronary angiogram shows only minimal non-obstructive coronary artery disease. What is the likely explanation for this patient's anginal symptoms? A. Small vessel coronary disease B. Syndrome X or Da Costa’s syndrome C. Coronary vasospasm D. Increased wall stress E. Left ventricular aneurysm
Correct Answer: D. Increased wall stress
96
Explanation: This patient has severe aortic valve stenosis on physical examination. This can cause angina due to increased myocardial oxygen demand from increased wall stress. Remember the Law of LaPlace:
97
An 18-year-old athletic man had been experiencing exertional chest pains typical for angina.  He suddenly collapsed during football practice and was found to be in ventricular fibrillation. He was successfully cardioverted. His vital signs stabilize. His cardiovascular physical examination is normal. His ECG is below: Enlarge  Which of the following is the most likely diagnosis? A. Hypertrophic obstructive cardiomyopathy B. Congenital coronary anomaly C. Coronary vasospasm D. Coronary embolus E. Brugada syndrome
Correct Answer: B. Congenital coronary anomaly
98
Explanation: There are many types of congenital variations of the coronary arteries. A majority of these are benign
however some anomalous coronary arteries can pass between the aorta and the pulmonary artery (known as an interarterial course) which can cause compression
99
A 78-year-old man presents to the emergency department with palpitations. He is afebrile with a heart rate of 140 beats per minute, blood pressure 120/80 mm Hg and respirations 22 per minute. Laboratory evaluation is normal. His ECG is below. What is the appropriate initial management strategy? Enlarge  A. Intravenous diltiazem B. Intravenous lidocaine C. Emergent direct current cardioversion D. Adenosine infusion
Correct Answer: A. Intravenous diltiazem
100
Explanation: Differentiating ventricular tachycardia from atrial fibrillation (a form of supraventricular tachycardia) can at time be difficult. This patient had a widened QRS complex with a left bundle branch block. When atrial fibrillation developed
a wide QRS complex tachycardia was seen. Because a ventricular tachycardia is a life-threatening wide complex QRS tachycardia
101
A 58-year-old man presents to the emergency department with chest pains. He is afebrile with a heart rate of 70 beats per minute, blood pressure 130/90 mm Hg, respirations 20 per minute. He is given appropriate therapy with aspirin, clopidogrel, unfractionated heparin and a beta-blocker. His ECG is below: Enlarge  Image: Learn the Heart The nearest hospital capable of percutaneous coronary intervention is 80 minutes away. Which of the following is the most appropriate course of action? A. Immediate transfer for primary percutaneous coronary intervention (PCI) B. Administration of a direct thrombin inhibitor C. Administration of fibrinolytic therapy D. Administration of fibrinolytic therapy and transfer for PCI (facilitated PCI)
Correct Answer: C. Administration of fibrinolytic therapy
102
Explanation: The decision regarding primary PCI vs. fibrinolytic therapy is important. Many major medical facilities have PCI capabilities because this is the treatment of choice for ST-elevation myocardial infarction. However
smaller hospitals or those located in rural areas may not. Those facilities frequently have capabilities to quickly transfer patients experiencing a ST-elevation myocardial infarction to a primary PCI facility. When there is no primary PCI available and transfer to a primary PCI facility is not able to be done in a timely fashion ― that is
103
Facilitated PCI refers to using fibrinolytic therapy to stabilize the patient while transport to a primary PCI facility is being arranged. This strategy receives a class IIb indication for high-risk patients with a low bleeding risk when primary PCI is not readily available. The patient in this case does not have any high-risk features (i.e. arrhythmia
shock
104
A 65-year-old man is post-operative day 2 from aortic valve replacement and coronary bypass surgery when he develops acute onset tachycardia with the below ECG tracing. He has mild palpitations but no other symptoms. He has been taking atenolol 100 mg orally daily. He is afebrile with a heart rate of 140 beats per minute, blood pressure 120/80 mm Hg and respirations 20 per minute. Laboratory evaluation is normal. What is the appropriate initial management strategy? Enlarge  A. Intravenous amiodarone B. Flecainide 300 mg by mouth once C. Emergent direct current cardioversion D. Transesophageal echocardiography, followed by elective direct current cardioversion
Correct Answer: A. Intravenous amiodarone
105
Explanation: Atrial fibrillation very commonly occurs after open heart surgery
in about 10% to 50% of cases. Amiodarone is safe and effective to convert patients to sinus rhythm in this setting. In a large majority of cases
106
A 68-year-old male who had delayed treatment of syphilis presents with generalized fatigue. He denies dyspnea, lower extremity edema or orthopnea. His blood pressure is 170/90 mm Hg and heart rate 80 bpm. A III/IV, short, early diastolic murmur is heard at the right upper sternal border. Systolic pulsation of the uvula and systolic capillary pulsations are seen upon light compression of the nail bed. Echocardiography confirms severe aortic valve regurgitation from a dilated aortic root. The ejection fraction is 60%. The left ventricular end-systolic dimension is 5.7 cm and the left ventricular end-diastolic dimension is 7.6 cm. Which of the following is the most appropriate course of action? A. Start nifedipine and repeat an echocardiogram in 6 months B. Start an ACE inhibitor and repeat an echocardiogram in 6 months C. Start a beta-blocker and repeat an echocardiogram in 6 months D. Surgical aortic valve replacement
Correct Answer: D. Surgical aortic valve replacement
107
Explanation: One of the indications for aortic valve replacement (AVR) is asymptomatic severe aortic regurgitation with evidence of pathologic remodeling: a left ventricular end-systolic dimension of greater than 50 mm (class 2a) or end-diastolic dimension of 65 mm (class 2 b)
as in this case.
108
According to the 2020 American College of
109
Cardiology/American Heart Association Guideline for the Management of Patients
110
with Valvular Heart Disease
AVR is indicated in patients with:
111
symptomatic severe aortic regurgitation (class I)
112
severe aortic regurgitation with no symptoms and left ventricular ejection
113
fraction ≤ 55% (class 1)
114
severe aortic regurgitation with no symptoms who are undergoing another cardiac
115
surgery
such as CABG
116
dilation of the aortic root or ascending aorta (class 1)
117
severe aortic regurgitation with no symptoms
a normal left ventricular ejection
118
fraction
but left ventricular end-systolic dimension greater than 50 mm
119
(class 2a)
120
severe aortic regurgitation with no symptoms
but progressive decrease in left
121
ventricular ejection fraction to < 55%-60% or increase in left
122
ventricular end-diastolic dimension to > 65 mm on at least three
123
studies (class 2b)
124
moderate aortic regurgitation who are undergoing another cardiac surgery
such as
125
CABG
mitral valve surgery or replacement of the ascending aorta (class
126
2a).
127
No medical therapy has been shown to prolong the need for
128
valve replacement
although decreasing the afterload with nifedipine or an ACE
129
inhibitor can improve symptoms.
130
Question 67/95 Flag A 78-year-old women presents to the emergency department with weakness and shortness of breath. The “indigestion” she has been having for 1 week is getting better. She is afebrile, with a heart rate of 120 beats per minute, respirations 26 per minute and blood pressure 80/40 mm Hg. Her physical examination reveals rales on lung examination and a loud V/VI holosystolic murmur at the left lower sternal border radiating throughout the precordium. Chest X-ray reveals pulmonary edema. Her ECG is below: Enlarge  Image: Learn the Heart Coronary angiography is done emergently, and her right coronary artery is completely occluded. Right heart catheterization shows normal V waves in the pulmonary capillary wedge pressure tracing. The oxygen saturation measured in the right atrium was 65%, in the right ventricle 88% and in the pulmonary artery 88%. The right atrial pressure is 20, right ventricle pressure 50/10 and pulmonary artery pressure 42/12. What is the correct diagnosis? A. Acute ventricular septal defect B. Acute left ventricular free wall rupture C. Acute mitral valve regurgitation D. Right ventricular infarction
Correct Answer: A. Acute ventricular septal defect
131
Explanation: This case is an example of an inferior ST segment elevation myocardial infarction complicated by an acute ventricular septal defect. When infarction of the interventricular septum occurs
this area can thin with the remodeling process and
132
With an acute ventricular septal defect
right heart catheterization will show an “oxygen step-up” between the right atrium and right ventricle or pulmonary artery
133
Question 70/95 Flag A 51-year-old male with a history of uncontrolled hypertension presents to the ED with acute onset substernal chest pain that is sharp in nature and radiates to the mid-back. He is short of breath and weak. His temperature is 99.0, blood pressure 80/30 mm Hg, heart rate 130 bpm and respirations 30 per minute. Physical examination reveals rales on lung exam and a short, III/IV early diastolic murmur at the right upper sternal border. Which of the following is the most likely diagnosis? A. Acute myocardial infarction B. Acute pulmonary embolus C. Acute aortic regurgitation D. Acute ventricular septal defect
Correct Answer: C. Acute aortic regurgitation
134
Explanation: This patient has an acute aortic dissection
resulting in acute severe aortic valve regurgitation and causing shock. This is apparent based on this description of the sudden onset chest pain
135
Question 74/95 Flag A 42-year-old healthy man presents with increasing dyspnea, lower extremity edema and fatigue. He denies chest pain, palpitations or syncope. Physical examination reveals a II/IV early diastolic short murmur with a systolic ejection click. There is also a II/IV diastolic murmur at the cardiac apex. Which of the following is the most likely finding on echocardiography? A. Aortic stenosis B. Aortic regurgitation C. Mitral valve stenosis D. Pulmonic valve regurgitation E. Atrial septal defect
Correct Answer: B. Aortic regurgitation
136
Explanation: Aortic regurgitation causes an early diastolic decrescendo murmur. The more severe the regurgitation
the shorter the murmur becomes because the aortic and left ventricular pressure equalize quickly. A systolic ejection click is indicative of a bicuspid aortic valve as the etiology.
137
Question 75/95 Flag A 72-year-old man with no cardiac history is admitted to the hospital with a low-grade fever and weakness. He has a history of leukemia and active chemotherapy treatment. He suddenly has the acute onset of palpitations and mild shortness of breath. His ECG reveals atrial fibrillation with an uncontrolled ventricular response. His respiratory rate is 20 per minute, blood pressure is 100/60 mm Hg, heart rate is 145 beats per minute and oxygen saturation is 98% on room air. IV diltiazem is administered to the patient and the heart rate remains 140 bpm  in atrial fibrillation; however, the blood pressure decreases to 80/40 mm Hg and the patient becomes dizzy and confused. A fever of 102.2 F develops. Laboratory evaluation is available, revealing a white blood cell count of 0.9 thousand/mm3. What is the next step in management for his atrial fibrillation? A. Emergent direct current cardioversion B. Intravenous diltiazem C. Intravenous metoprolol D. Intravenous amiodarone E. Intravenous digoxin
Correct Answer: D. Intravenous amiodarone
138
Explanation: It is important to remember to think of the etiology of the atrial fibrillation when determining management strategies. This patient appears to be experiencing severe sepsis (perhaps septic shock) related to a neutropenic fever. Sepsis is a common cause of atrial fibrillation
and treating the uncontrolled ventricular rate with non-dihydropyridine calcium channel blockers (diltiazem
139
Question 77/95 Flag A 72-year-old man with no cardiac history is admitted to the hospital with a low-grade fever and weakness. He has a history of leukemia and active chemotherapy treatment. He suddenly has the acute onset of palpitations and mild shortness of breath. His ECG reveals atrial fibrillation with an uncontrolled ventricular response. His respiratory rate is 20 per minute, blood pressure is 100/60 mm Hg, heart rate is 145 beats per minute and oxygen saturation is 98% on room air. What is the appropriate next step in management? A. Emergent direct current cardioversion B. Intravenous diltiazem or metoprolol C. Intravenous digoxin D. Intravenous amiodarone E. Intravenous procainamide
Correct Answer: B. Intravenous diltiazem or metoprolol
140
Explanation: In patients with uncontrolled ventricular rates
atrioventricular blocking agents are important to slow the heart rate and decrease symptoms from atrial fibrillation. Intravenous nondihydropyridine calcium channel blockers (diltiazem
141
Question 78/95 Flag A 42-year-old man with no past cardiovascular medical history comes to the emergency department with a complaint of shortness of breath and lower extremity edema for 1 month. He is afebrile with a heart rate of 160 beats per minute, respirations 20 per minute, blood pressure 140/90 mm Hg and oxygen saturation 92% on room air. His jugular venous pressure is markedly elevated. Lung exam reveals diffuse rales. A II/VI systolic ejection murmur is appreciated at the right upper sternal border. An S3 heart sound is present, and the point of maximal impulse is laterally displaced; no S4 is heard. There is 3+ pitting edema up to his knees. His ECG confirms atrial fibrillation with an uncontrolled ventricular rate. What is the most appropriate initial management? A. Intravenous amiodarone B. Intravenous ibutilide C. Intravenous diltiazem or metoprolol D. Intravenous digoxin E. Emergent direct current cardioversion
Correct Answer: C. Intravenous diltiazem or metoprolol
142
Explanation: The patient’s heart rate is significantly elevated in the setting of atrial fibrillation
and he is hemodynamically stable; thus
143
A 52-year-old man presents for a routine physical examination, which reveals a III/VI systolic ejection murmur with a systolic ejection click and a soft S2 heart sound. He has no dyspnea or chest pains and has never had a syncopal episode. Echocardiography reveals aortic stenosis with a mean pressure gradient of 35 mm Hg with an aortic valve area of 1.2 cm2. The aortic root was measured at 5.2 cm. Which of the following is the most likely etiology? A. Senile calcific degeneration B. Bicuspid aortic valve C. Rheumatic valvular disease D. Fabry’s disease
Correct Answer: B. Bicuspid aortic valve
144
Explanation: A bicuspid aortic valve is the most common cardiac congenital anomaly
is considered a connective tissue disorder and is the most common cause of aortic stenosis in patients aged younger than 70 years old. The aortic valve is normally tricuspid or trileaflet. When two of the cusps are fused together
145
An asymptomatic 45-year-old man is being seen for a routine physical examination. His blood pressure is 110 mm Hg systolic/85 mm Hg diastolic. A IV/VI mid-systolic murmur is appreciated at the cardiac apex, radiating to the axilla, occurring after a mid-systolic click. The murmur becomes louder with transient arterial occlusion, or TAO, and moves earlier in systole with standing from a squatting position. Assuming the condition is severe upon echocardiography, with an ejection fraction of 65%, what is the most appropriate course of action? A. Start an ACE inhibitor, and repeat the echocardiogram in 1 year B. Start nifedipine, and repeat the echocardiogram in 1 year C. Mitral valve repair D. Mitral valve replacement
Correct Answer: C. Mitral valve repair
146
Explanation: This patient has severe mitral valve regurgitation
from mitral valve prolapse
147
Question 91/95 Flag A 32-year-old man presents with dyspnea and hemoptysis. He is afebrile, with a heart rate of 100 beats per minute, blood pressure is 120 mm Hg systolic/80 mm Hg diastolic and respirations are 22 per minute. His cardiac physical examination reveals a soft, II/IV early diastolic murmur at the cardiac apex. A bronchoscopy is negative for any lesion or malignancy. Echocardiography will likely reveal: A. Aortic regurgitation (with Austin-Flint phenomenon) B. Patent ductus arteriosis C. Ventricular septal defect D. Atrial septal defect E. Mitral stenosis
Correct Answer: E. Mitral stenosis
148
Explanation: “Pulmonary apoplexy” refers to hemoptysis that occurs from rupture of a bronchial vein due to severe mitral valve stenosis
causing pulmonary venous hypertension.